Sie sind auf Seite 1von 65

PRE GATE - 2018

Electronics & Communication Engineering (EC)


The GA section consists of 10 questions. Questions 1 to 5 are of 1 mark each, and
Questions 6 to 10 are of 2 marks each.
01. Choose the most appropriate word from the options given below to complete the following
sentence:
If I had known that you were coming, I ______________ you at the airport.

(A) would have met (B) shall have met (C) would meet (D) would have meeting
Ans: (A)
Exp: Conditional tense type three should be if clause (had+v3) and the main clause (would+have+v3)

02. In the following sentence certain parts are underlined and marked P, Q and R. One of the
parts may contain error or may not be acceptable in standard written communication.
Select the part containing an error. Choose D as your answer if there is no error.
They found that five prisoners escaped the previous night. No error
(P) (Q) (R) (D)
(A) P (B) Q
(C) R (D) No error
Ans: (B)
Exp: In part 'Q' it should be 'prisoners had escaped' because the earlier action of the past should be in
past perfect tense (had + v3).

03. The question below consists of a pair of related words followed by four pairs of words. Select
the pair that best expresses the relation in the original pair.
LAVA : VOLCANO ::
(A) ice : glass
(B) cascade : precipice
(C) steam : geyser
(D) avalanche : ice
Ans: (C)
Exp: (DEFINING CHARACTERISTIC) A volcano gives out lava (meaning molten rock); a geyser
(meaning a natural hot spring) gives out steam.

ACE Engineering Academy Hyderabad|Delhi|Bhopal|Pune|Bhubaneswar| Lucknow|Patna|Bengaluru|Chennai|Vijayawada|Vizag |Tirupati | Kukatpally| Kolkata



: 2 : ECE

04. If “JUICE” is coded as “19-41-17-5-9”, then “TOY” will be coded as
(A) 39-29-49 (B) 41-31-51
(C) 13-23-3 (D) 15-25-5
Ans: (A)
Sol:

AS,
10 21 9 3 5
J U I C E

102 212 92 32 52


–1 –1 –1 –1 –1
19 41 17 5 9

Same as,
20 15 25
T O Y

202 152 252


–1 –1 –1
39 29 49

But in option (B)


20 15 25
T O Y

202 152 252


+1 +1 –1
41 31 51

It is not in that code


Option (C) and (D) are not correct

ACE Engineering Academy Hyderabad|Delhi|Bhopal|Pune|Bhubaneswar| Lucknow|Patna|Bengaluru|Chennai|Vijayawada|Vizag |Tirupati | Kukatpally| Kolkata



: 3 : ECE

05. Fill in the blanks with an appropriate idiom
Let us have your terms ____________
(A) through thick and thin
(B) in black and white
(C) ins and outs
(D) at cross-purposes
Ans: (B)
DLOA: through thick and thin means under all conditions
DLOB: correct answer - in black and white means in writing
DLOC: ins and outs means full details
DLOD: at cross-purposes means misunderstand each other
So the right option is 'B'

06. Statement: These apples are too expensive to be bad.


Which of the following can be logically inferred from the above statement?
I. The higher the selling price, the superior is the quality of the commodity.
II. when apples are in short supply, the prices go up.
(A) only I (B) only II
(C) I & II (D) None of the above
Ans: (A)
Exp: The second conclusion is irrelevant. The first is the meaning of the given statement. 'Too
expensive to be bad' means that it can't be bad because it is expensive.

07. A postman walked 7 km north from the post office to reach Mr. Singh’s house. He then took a
left turn and walked 4 km to reach Mr. Kumar’s house. He then took a right turn and walked 3
km to reach Mr. Sharma’s house. The distance between Mr. Sharma’s and Mr. Singh’s house is
______
(A) 5 km (B) 6 km
(C) 4 km (D) 7 km

ACE Engineering Academy Hyderabad|Delhi|Bhopal|Pune|Bhubaneswar| Lucknow|Patna|Bengaluru|Chennai|Vijayawada|Vizag |Tirupati | Kukatpally| Kolkata



: 4 : ECE

Ans: (A)
Sol:
Let the postman started from the point P which denote post office.

C (Mr. Sharma’s house)


N
3km

W E
B A (Mr. Singh’s house)
(Mr. Kumar’s house) 4 km
7 km
S

P
(Post office) starting point
We have to find the distance between A and C so, applying Pythagoras theorem,
AC2 = AB2 +BC2 = 16 + 9
AC2 = 25
AC = 25 = 5 cm
So, distance between Mr. Sharma’s and Mr. Singh’s house is 5 km.

ACE Engineering Academy Hyderabad|Delhi|Bhopal|Pune|Bhubaneswar| Lucknow|Patna|Bengaluru|Chennai|Vijayawada|Vizag |Tirupati | Kukatpally| Kolkata



: 5 : ECE

08. How many 3-digit even number can be formed from the digits 1, 2, 3, 4, 5, 6, if the digits can be
repeated?
(A) 216 (B) 108 (C) 96 (D) 54
Ans: (B)
Sol:

Hundred Tens Unit place


We know that, a number is called even, if its unit's place is occupied by an even digit (i.e) 2,4,6
So, for unit place, we have 3 options
For tens place, there are 6 options
For hundred place, there are 6 options
[ repetition allowed]
 Total number of ways in which 3 digit even numbers can be formed = 366 = 108
09. Examine the information given below
All the leaders are believable
Some believable persons are intelligent

Which of the following is a valid conclusion regarding the above arguments?


(A) All the leaders are intelligent (B) some leaders are believable
(C) All the intelligent persons are leaders (D) some believable persons are leaders
Ans: (D)
Sol: The given statements can be represented as
Case – I
Believable

Intelligent

(i)
(or) Leader

Case – II
ACE Engineering Academy Hyderabad|Delhi|Bhopal|Pune|Bhubaneswar| Lucknow|Patna|Bengaluru|Chennai|Vijayawada|Vizag |Tirupati | Kukatpally| Kolkata

: 6 : ECE

Believable

Intelligent

(ii)
Leader

From above two diagrams


Hence, option (D) is correct

10. A shopkeeper sells note books at the rate of ₹457 each and earns a commission of 4%. He also

sells pencil boxes at the rate of ₹80 each and earns a commission of 20%. How much amount of

commission will he earn in two weeks, if he sells 10 note books and 6 pencil boxes a day?

(A) ₹1956 (B) ₹1586

(C) ₹1496 (D) None of these

Ans: (D)
Sol: S.P of the note book = ₹457
4  457
 Commission on one note book = ₹
100
10  4  457
and commission on 10 note books = = ₹ 182.80
100
and S.P of the pencil box = ₹80
80  20
 Commission on one pencil box = ₹
100
80  20  6
Commission on 6 pencil boxes = = ₹96
100
Hence, total commission of one day = (182.80+96) = ₹ 278.80
Thus, total commission of two weeks = 278.80 14 = ₹3903.20
Option (A) is not correct
Option (B) is not correct
Option (C) is not correct
 None of these is the answer

ACE Engineering Academy Hyderabad|Delhi|Bhopal|Pune|Bhubaneswar| Lucknow|Patna|Bengaluru|Chennai|Vijayawada|Vizag |Tirupati | Kukatpally| Kolkata



: 7 : ECE

         The  subject  specific  section  of EC consists  of 55 questions,  out  of  which  question 

         numbers 1 to 25 are of 1 mark each, while question numbers 26 to 55 are of 2 marks each 

01. VG is the voltage applied across a MOS capacitor with a P-type Si substrate and a doping density
of 1017 cm–3. Thickness of the oxide layer is 3.5nm. If VG = VTH + 1V, where VTH is the
threshold voltage of the MOS capacitor. The areal inversion charge density is _____ (in
C/cm2). [Assume ox = 3.510–13 F/cm]
Ans: –1
Sol: VG = VTH + 1V
VG – VTH = 1V  MOS capacitor is operating in inversion mode.
Q INV  C ox VG  VTH 

 ox
 VG  VTH 
t ox

3.5  10 13
 (1V)
3.5  10 7
= –10–6 C/cm2
= –1C/cm2

02. A parallel circuit consists of a 4F capacitor and a coil whose inductance and resistance are 2mH
& 10 respectively is connected to the output of a transistor acts as a current source as shown in
figure. Determine the value of Qcoil at resonance

Transistor

Is Rs coil C

(A) 5 (B) 3
(C) 1 (D) 2
ACE Engineering Academy Hyderabad|Delhi|Bhopal|Pune|Bhubaneswar| Lucknow|Patna|Bengaluru|Chennai|Vijayawada|Vizag |Tirupati | Kukatpally| Kolkata

: 8 : ECE

Ans: (D)
2 2
1 R 1  10 
Sol: 0      0  3 6
 3 
LC  L  2  10  4  10  2  10 

0  10 3 125  25  10 4 rad / sec

0 L 10 4  2  10 3
Q coil   2
R 10

Distractor Logic

1 1 105
DLOA: 0    rad/sec
LC 2  10 3  4  10 6 80

0 L 10 5  2  10 3 200 20 5
Q coil       5
R 80  10 80  10 80 5

DLOB: Qcoil = 3 is not related to any solution

2
R R 10
DLOC: 0     
L L 2  10 3

0 L 2  10 3  10
Q coil   1
R 10  2  10 3

DLOD: Correct option

03. Find the output produced by a Miller integrator in response to an input pulse of 1 V height and
1ms width. The op-amp is specified to saturate at  13V
t=0 10nF
Vin(V)

10k 1

V0
+
Vin +
– t (ms)
0 1

ACE Engineering Academy Hyderabad|Delhi|Bhopal|Pune|Bhubaneswar| Lucknow|Patna|Bengaluru|Chennai|Vijayawada|Vizag |Tirupati | Kukatpally| Kolkata



: 9 : ECE

(A) V0 1ms (B)


t V0 1 ms
0 t

–10V
–10V
–13V

(C) V0 (D) V0
10V +13V
+10V

0V 1ms t 0V 1ms t

Ans: (A)
Sol:
10nF
+ Vc –
10k

(0V) V0
I +

1V

1
I A
10k V0 1ms
0 t
1
C 
V0 = –Vc  Idt

I –10V
  t
C
 1 
 ( t )
 10k 

10nF
 104 t
At, t = 1ms
V0 = –10410–3 = –10V
ACE Engineering Academy Hyderabad|Delhi|Bhopal|Pune|Bhubaneswar| Lucknow|Patna|Bengaluru|Chennai|Vijayawada|Vizag |Tirupati | Kukatpally| Kolkata

: 10 : ECE

Distractor Logic
DLOA: Option (A) is correct
DLOB: If the student forgets that the input is a pulse and assumes the ramp is increasing upto the
saturation level then option (B) will match which is wrong
DLOC: The student calculates the capacitor voltage and forgets the phase shift of V0
DLOD: If the student forgets that the input is a pulse and assumes the ramp is increasing upto the
saturation level and also neglecting the phase shift

04. Let X(k) be the even length N-point DFT of a real signal x(n). Then the number of DFT samples
that will be always real is ____________
Ans: 2
N 1 j 2 nk

Sol: DFT of x(n) is X(k )   x (n )e N

n 0

N 1
For even length ‘N’, X(0)   x (n ) → always real
n 0

 N  N1
X    x (n ) (1) n → always real
 2  n 0
For even length ‘N’, only 2 samples are real

sin x
05. The mean value of Rolle's theorem for the function f ( x )  in [0, ] is _____ (in rad)
ex
Ans: 0.785 (Range 0.78 to 0.79)
Sol: By Rolle’s theorem,
f (c)  0

e  c cos c  sin c   0
 cos c – sin c = 0
sin c
 1
cos c
 tan c = 1

c  (0, )
4
ACE Engineering Academy Hyderabad|Delhi|Bhopal|Pune|Bhubaneswar| Lucknow|Patna|Bengaluru|Chennai|Vijayawada|Vizag |Tirupati | Kukatpally| Kolkata

: 11 : ECE

06. What is the value of RL and power, when maximum power transfer to RL for the following circuit

20V 40
120 RL
20 60

(A) 12 , 2.5 W (B) 10 , 2.5 W


(C) 12 , 10 W (D) 10 , 10 W
Ans: (B)
Sol: Across RL Thevenin Equivalent
20
+

20V 40
120 VOC
60

By Voltage Division rule


20  20
VOC   10V (40//60//120 = 20)
20  20
20
For Rth(V S.C)
Rth = (20//40//60//120)
40
= 10 = RL 120  Rth
60

Pmax 
Vth2

10  2.5W
2

4  R th 4  10

Distractor Logic
DLOA: 12 , 2.5 W
Rth is wrong,
 12 
= 10 
Vth2 (10) 2
Pmax    2.5 W is correct
4R th 4  10
ACE Engineering Academy Hyderabad|Delhi|Bhopal|Pune|Bhubaneswar| Lucknow|Patna|Bengaluru|Chennai|Vijayawada|Vizag |Tirupati | Kukatpally| Kolkata

: 12 : ECE

DLOB: Correct option

DLOC: 12 , 10 W
Rth is wrong,
Vth2 (10) 2
Pmax    10 W is wrong
R th 10

DLOD: 10 , 10 W
Rth is correct (60//120//40//20 = 10 )
Vth2 (10) 2
Pmax    10 W is wrong
R th 10

07. An amplifier designed using a MOSFET has the transfer characteristics V0 = 10–5(Vi–2)2, where
Vi and V0 are in volts. The transfer characteristic applies for 2  Vi  V0+2 and V0 is positive.
The value of small signal voltage gain at a bias point where V0 = 5V is ______
Ans: –10
Sol: To obtain V0 = 5V, we bias Vi = 3V [5 = 10 – 5(Vi – 2)2, Vi = 3]
V0
Small signal voltage gain (Av) =  5  2(Vi  2)
Vi
= –10(3–2)
= –10(V/V)

 3 1 
08. If A    then A100 = ?
 2  3
(A) 1150 I (B) 11100 I

(C) 1150 A (D) 11100 A


Ans: (A)
Sol: The characteristic equation is A  I  2  0.  11  0

 2  11  0

ACE Engineering Academy Hyderabad|Delhi|Bhopal|Pune|Bhubaneswar| Lucknow|Patna|Bengaluru|Chennai|Vijayawada|Vizag |Tirupati | Kukatpally| Kolkata



: 13 : ECE

By Cayley-Hamilton theorem
A 2  (11)I  0
A 2  (11) I
A100  A 2   50
 11 I   1150 I
50

09. If X, Y are two random variables and joint pdf of X and Y is given by
x 2  y2
f XY ( x, y)  ; x2 + y2 < b
8
= zero ; otherwise
The magnitude of ‘b’ is ___________
Ans: 4
Sol: f XY ( x, y) dx dy  1

Converting the rectangular co-ordinates into circular co-ordinates we get,


x 2  y2  b , dx dy  rdr d
Radius of circle is b
b 2
r2
 
r 0
 8 r dr d  1
 0

 b2 = 16
b=4

10. Consider a cylinder of radius R = 10cm and length L = 5m that is non-uniformly charged with
charge density (r) = 10–3 r2 C/m3. The cylinder rotates with uniform angular velocity ‘’ around
the z-axis, which is also the center axis of the cylinder as shown in figure. Then the current ‘I’ at
R
r is _________. (given = 103rad/sec)
2
z
 = 103 rad/sec

L = 5m

ACE Engineering Academy Hyderabad|Delhi|Bhopal|Pune|Bhubaneswar| Lucknow|Patna|Bengaluru|Chennai|Vijayawada|Vizag |Tirupati | Kukatpally| Kolkata



: 14 : ECE

(A) 0.21 mA (B) –7.81 A
(C) –0.21 mA (D) 7.81 A
Ans: (D)
Sol: J (r )  (r ) v(r )  (r )   r  (r ) â z  râ r   (r )  r â 
r L r L r r
 I( r )    (r)  r â
r 0 z 0
 . dr dz â     (r)  r dr dz
r  0 z 0
 L  (r ) r dr  L  10 3 r 3 dr
r 0 r 0

r4 3
I(r )  L  10
4
R
at r 
2

 R 3
I r    L  10 
R4 3
 10  5 10 
3 
10  10  2 

5
 10  4 = 0.078125  10–4
4

 2 4  16 64 64
= 7.8125 A.

Distractor Logic
DLOA: If they consider linear velocity & angular velocity as same then v =  = 103 rad/sec
Then J (r )  (r ) â 
r r
r3
 I(r )  L  (r ) dr  L  10 3 r 2 dr  L  10 3
r 0 r 0
3

R
at r   5cm
2

 I(r  5cm)  10 3  5 
5  10  2 3
 10 3 
5 125  10 6
3 3
= 0.208mA
 0.21mA

DLOB: In option (D)  If they consider direction of angular velocity as  â z 

v(r )    r  `r  â  

Then J (r )  (r ) r  â  

ACE Engineering Academy Hyderabad|Delhi|Bhopal|Pune|Bhubaneswar| Lucknow|Patna|Bengaluru|Chennai|Vijayawada|Vizag |Tirupati | Kukatpally| Kolkata



: 15 : ECE

 R
Then we get I r   (i.e)
 2
I(r = 5cm) = –7.81A

DLOC: In option (A) if they consider


J (r )  (r )  â  

Then we get
I(r = 5cm) = –0.208mA  –0.21mA

DLOD: Correct Option

11. The transfer function to the given block diagram is


1 S
R(S) + 1  0.5S + 5 C(S)
– + S2
10

60S  110 60S  110


(A) (B)
S  2S2  120S  220
3
S  2S2  120S  220
3

5S  5 S3  2S2  300S  55
(C) (D)
 9.5S  9S2  10S  10
3
0.5S3  S2
Ans: (B)
 1 S   11  6S 
Sol: Parallel block combination   10   
1  0.5S   1  0.5S 
5 5  11  6S   55  30S 
Above blocks are series with  2  
S 2
S  1  0.5S   S2 1  0.5S 

55  30S
 Simplified feedback 
0.5S  S2  255  30S
3

CS 30S  55

R S 0.5S  S2  60S  110
3

60S  110

S  2S2  120S  220
3

ACE Engineering Academy Hyderabad|Delhi|Bhopal|Pune|Bhubaneswar| Lucknow|Patna|Bengaluru|Chennai|Vijayawada|Vizag |Tirupati | Kukatpally| Kolkata



: 16 : ECE

Distractor Logic
DLOA: If the feedback loop is consider as positive, then option A is correct.
DLOB: Correct answer:
DLOC: If block “Gain 10” is consider as a feedback, then option C is correct.
DLOD: If block “Gain 2” is consider as parallel, then option D is correct

12. A received signal is 1mV for Tb seconds time interval with equal probability. The signal is
corrupted by AWGN having two sided PSD of 10–10 (W/Hz). The receiver integrates the signal
plus noise synchronously for Tb seconds duration and decodes the signal by comparing the
integrator output with ZERO [Threshold value is ZERO in optimum receiver]. The maximum
signalling rate in bps such that the value of Pe = 10–4 is ___________
NOTE: Q [ 3.7] = 10–4
Ans: 730 (Range: 725 to 735)
Sol: The given receiver indicates the integrate and dump receiver optimized for getting minimum Pe.
The value of Pe is given by
 Ed 
Pe  Q 

 2N0 

Ed is Energy difference of two signals for ‘Tb’ duration at Receiver input.

S1(t) S2(t)

1mV 0 Tb
t

0 t –1mV
Tb

Tb

 2mV 
2
 Ed  dt  4  10 6  Tb
0

N0
 10 10 W / Hz 
2
ACE Engineering Academy Hyderabad|Delhi|Bhopal|Pune|Bhubaneswar| Lucknow|Patna|Bengaluru|Chennai|Vijayawada|Vizag |Tirupati | Kukatpally| Kolkata

: 17 : ECE

 4  10 6  T 
 Pe  Q  10 
b
10 4
 2  2  10 
 
Q Tb  104 10 4 [ given Q(3.7) = 10–4 ]
 T  10   3.7
b
4

1
 rb = = 730.46 (bps)
Tb

13. The delay produced by the following 8085 microprocessor program is _______ (in terms of
number of T-states)
T -States
MVI A, 25H  7T
MVI B, 10H  7T
NOP  4T
NOP  4T
LOOP: SUB B  4T
XRA A  4T
JNZ LOOP  7T/10T
(false)/(true)
Ans: 37
Sol: MVI A, 25H  7T
MVI B, 10H  7T
NOP  4T
NOP  4T
LOOP: SUB B  4T
XRA A  4T
JNZ LOOP  7T/10T
(false)/(true)
XRA A makes Z = 1. Hence the condition is false for JNZ loop instruction.
Thus number of T-states are
7T + 7T + 4(4T) + 7T = 37T
ACE Engineering Academy Hyderabad|Delhi|Bhopal|Pune|Bhubaneswar| Lucknow|Patna|Bengaluru|Chennai|Vijayawada|Vizag |Tirupati | Kukatpally| Kolkata

: 18 : ECE

ACE Engineering Academy Hyderabad|Delhi|Bhopal|Pune|Bhubaneswar| Lucknow|Patna|Bengaluru|Chennai|Vijayawada|Vizag |Tirupati | Kukatpally| Kolkata



: 19 : ECE

14. For a signal x(t), the spectrum is shown in figure. Find the signal value at t = 0 sec.
X() X()

2 2
–2

 0 2
–2 0 2 

2

16 8
(A) (B)
 2
16
(C) 2 (D)
2
Ans: (B)
Sol: X ()  X () e jX ( )

j 
X()  2e 4
2 2
Inverse Fourier Transform is given by
1  jt
x (t )   X() e d
2  
2
 j   j
2
j
2

1
2
j  
2 e 4 
e e 4
4

2 2
x (0)  2e 4
d    
2  j   2
j
 4  2 4
2j 8
  2
 2

j
4

Distractor Logic
1
DLOA: In the inverse Fourier Transform expression if we neglect , we will get answer (A)
2
DLOB: Correct option
DLOC: You may feel peak amplitude of the spectrum itself as x(0)  answer (C)
1 1
DLOD: In the inverse Fourier Transform expression instead of , if we take will get answer (D)
2 
ACE Engineering Academy Hyderabad|Delhi|Bhopal|Pune|Bhubaneswar| Lucknow|Patna|Bengaluru|Chennai|Vijayawada|Vizag |Tirupati | Kukatpally| Kolkata

: 20 : ECE

15. For the block diagram shown below, the magnitude of sensitivity ST at  = 0.1 rad/sec for  = 2

is ___________.

R(S) +  1 C(S)
– S S 1

Ans: 0.05 (Range: 0.04 to 0.06)


 
Sol: CLTF, T   2
SS  1   S  S  

 T / T     T  S2  S
ST         2
  /    T    S  S  

  2  j  0.01  j0.1  0.01  j0.1


S T
    0.05
   j   2  0.01  j0.1
 0.1
2
2
1.99  j0.1

16. An air filled rectangular waveguide of dimension ab (a > b), propagated in dominant mode has
a guide wavelength of 4cm for a frequency of 9GHz. The value of ‘a’ (in cm) of the wave guide
is _____.
Ans: 3 (Range: 2.8 to 3.2)
Sol: g = 4cm, f = 9GHz, m = 1, n = 0
c 3  1010
Wavelength     3.33cm
f 9  109
 3.33 (3.33) 2
Guided wavelength,  g   4  16  2
  
2
 3.33 
2
 3.33 
1    1    1   
 c  
 c 
 c 
c = 6.01cm
2 2a
Cut off wavelength,  c  
2
m n
2 m
   
 a  b
 c 6.01
a    3 cm
2 2
a  3 cm
ACE Engineering Academy Hyderabad|Delhi|Bhopal|Pune|Bhubaneswar| Lucknow|Patna|Bengaluru|Chennai|Vijayawada|Vizag |Tirupati | Kukatpally| Kolkata

: 21 : ECE

17. Implement f ( x1 , x 2 , x 3 , x 4 )  x1 x 3 x 4  x1 x 3 x 4  x 2 x 3 x 4 using a 8:1 MUX and x4, x 4 are
available.
When x1 x2 x3 are given as selection lines
(A) I0 = 0, I1 = 1, I 2  x 4 , I3 = 1 (B) I0 = 1, I1 = x 4 , I 2  x 4 , I3 = 0

I 4  x 4 , I5  x 4 , I 6  0 , I7  x 4 I4  0 , I5  x 4 , I6  0 , I7  x 4

(C) I0 = 1, I1  x 4 , I2 = x4, I3 = 0 (D) I0 = 1, I1 = 0, I2 = x4, I3 = 0


I 4  x 4 , I5  x 4 , I 6  0 , I7  x 4 I 4  x 4 , I5  x 4 , I6  0 , I7  x 4

Ans: (D)
Sol: f ( x1 , x 2 , x 3 , x 4 ) = x 1 x 2  x 2  x 3 x 4  x 1 x 2  x 2  x 3 x 4  x 1  x 1  x 2 x 3 x 4

f ( x1 , x 2 , x 3 , x 4 ) = x 1x 2 x 3 x 4  x1x 2 x 3 x 4  x1x 2 x 3 x 4  x1x 2 x 3 x 4  x1x 2 x 3 x 4  x1x 2 x 3 x 4

m(14, 10, 5, 1, 8, 0)
m(0, 1, 5, 8, 10, 14) 1 I0
0 I1
x4 I2 8:1
0 I3 MUX
x4 I4
x4 I5
0 I6
x4 I 7 s2 s1 s0

x1 x2 x3

x1x2x3
x4 000 001 010 011 100 101 110 111
0 1 0 0 0 1 1 0 1

1 1 0 1 0 0 0 0 0

1 0 x4 0 x4 x4 0 x4

Distractor Logic
DLOA: If consider the minterms as maxterms i.e., f = M(0, 1, 5, 8, 10, 14)
DLOB: Even though they have taken x1 as MSB, they connect selection lines s2s1s0 as x3x2x1
DLOC: If consider selection lines s2s1s0 as x3x2x1 respectively i.e., considering x4 as MSB
DLOD: Correct option

ACE Engineering Academy Hyderabad|Delhi|Bhopal|Pune|Bhubaneswar| Lucknow|Patna|Bengaluru|Chennai|Vijayawada|Vizag |Tirupati | Kukatpally| Kolkata



: 22 : ECE

18. The circuit shows an ideal voltage amplifier with a gain of 2(V/V). Find the equivalent input
resistance (Rin) ?
R

+2
R

Rin
(A)  (B) –R
R
(C) R (D) 
4
Ans: (A)
Sol: The circuit offers negative resistance
R
By considering Miller’s effect the input resistance of the Op-Amp is given by
1 A
R

R
 R
1 2
Now the input resistance of the given circuit is

R –R
R (R )
R in   
R R Rin

Distractor Logic
DLOA: Correct answer
DLOB: If the student forgets the parallel resistance R
DLOC: Student assumes unity gain and applies miller effect formula
R
R Miller   
0
R// = R

ACE Engineering Academy Hyderabad|Delhi|Bhopal|Pune|Bhubaneswar| Lucknow|Patna|Bengaluru|Chennai|Vijayawada|Vizag |Tirupati | Kukatpally| Kolkata



: 23 : ECE

DLOD: If student assumes gain is –2
R R R
 
1 A 1 2 3
R R
R // 
3 4

19. f1 and f2 functions are given as


f1 (A, B, C, D)  M (1, 3, 5,11,15)
f 2 (A, B, C, D)  M (1, 3, 5, 7, 8, 9,11)
Then find ‘f3’ expression where f3 = f1 + f2

(A)  M (1, 3, 5, 7, 8, 9,11,15) (B)  m(1, 3, 5,11)


(C) M (1, 3, 5,11) (D)  m(1, 3, 5, 7, 8, 9,11,15)
Ans: (C)
Sol: f1 (A, B, C, D)   m(0, 2, 4, 6, 7, 8, 9,10,12,13,14)

f 2 (A, B, C, D)   m(0, 2, 4, 6, 10,12,13,14,15)

f 3  f1  f 2
The function f3 consists of all the min-terms of function f1 and f2
f3   m(0, 2, 4, 6, 7, 8, 9,10,12,13,14,15)

f3 = M (1, 3, 5,11)

Distractor Logic

DLOA: If we directly apply OR to maxterms

DLOB: If we consider maxterms of final output as minterms

DLOC: Correct option

DLOD: If we directly apply OR to f1 and f2 and considering the output as in minterms

ACE Engineering Academy Hyderabad|Delhi|Bhopal|Pune|Bhubaneswar| Lucknow|Patna|Bengaluru|Chennai|Vijayawada|Vizag |Tirupati | Kukatpally| Kolkata



: 24 : ECE

cos z
20. The value of the integral  z  dz where c is |z–1| = 3 is
c

(A) –2i (B) 2i


(C) i (D) –i

Ans: (A)
Sol: The singular point z =  = 3.14 lies inside the circle C: | z –1| = 3
cos z
 z  dz = 2if()
c
where f(z) = cos z

= 2icos()
= –2i

ACE Engineering Academy Hyderabad|Delhi|Bhopal|Pune|Bhubaneswar| Lucknow|Patna|Bengaluru|Chennai|Vijayawada|Vizag |Tirupati | Kukatpally| Kolkata



: 25 : ECE

21. For a Gaussian Random variable ‘X’ the following data is given:  X  0 ,  X  1 .

What is P x  2

(A) Q(2) (B) 2Q(2)


(C) 1 – Q(2) (D) 1 – 2Q(2)
Ans: (B)
Sol:

 x
–2 x=0 2

 2  0
Px  2  1  Q   1  Q(2)  1  1  Q(2)  Q(2)
 1 
Px  2  Q(2)
 P | x |  2  Px  2  Px  2

P | x | 2  2Q(2)

Distractor Logic

DLOA: Is wrong answer


If student considers P[x > 2] instead of P[|x| > 2]
Then, P[x > 2] = Q(2)

DLOB: Is correct answer

DLOC: Is wrong answer


If student misinterprets as considering P[x > 2] = 1 – P[x  2]
= 1 – Q(2)
DLOD: Is wrong answer
If student does not use Q(–x) = 1– Q(x)
Then he may end up with 1 – 2Q(2)

ACE Engineering Academy Hyderabad|Delhi|Bhopal|Pune|Bhubaneswar| Lucknow|Patna|Bengaluru|Chennai|Vijayawada|Vizag |Tirupati | Kukatpally| Kolkata



: 26 : ECE

22. The 4-point DFT of x(n) is X(k) = {1, 2, 3, 4}. Find the DFT of the signal x(n+6).

(A) {1, 2, 3, 4} (B) {4, 3, 2, 1}


(C) {1, –2, 3, –4} \(D) {–1, 2, –3, 4}
Ans: (C)
j2 k

Sol: Circular shift property of DFT is x (n  n 0 ) N  W kn 0
N X(k ) . Where W  e k
N
N

Period of the given signal x(n) is N = 4


 N  DFT
 1 X(k )
k
x (n  6)  x (n  2)  x n   
 2 
So, DFT of x(n+6) = {1, –2, 3, –4}

Distractor Logic

DLOA: If you feel shifting doesn’t affect, then the DFT of x(n+6) is X(k). So, answer is (A)

DLOB: If you feel after circular shifting, the answer may be flipped version of X[k]  answer is (B)

DLOC: Correct option

DLOD: While doing the mistake in simplification of (–1)kX(k). Then option (D) is correct

23. Suppose ‘X’ is a poisson random variable and E(X2) = 6 then P(X  1.2) is

(A) 2e–2
(B) 3e–3
(C) 3e–2
(D) 2e–3
Ans: (C)
Sol: V(X) = E(X2) – (E(X))2
 = 6 – 2 where  is the mean of poisson distribution
 2 +  –6 = 0
=2
Thus P(X1.2) = P(X=0) + P(X=1) = e–2 + 2e–2 = 3e–2

ACE Engineering Academy Hyderabad|Delhi|Bhopal|Pune|Bhubaneswar| Lucknow|Patna|Bengaluru|Chennai|Vijayawada|Vizag |Tirupati | Kukatpally| Kolkata



: 27 : ECE

24. The system and its pole locations are shown below. The value of K is ______.

R(S) + K C(S)
– 
S  2S  4 S2  6S  25 

Ans: 666.25 (Range: 664 to 670)


Sol: CE  1  G S  0
 S4 + 12S3 +69S2 + 198S + 200 + K = 0

S4 1 69 K + 200

S3 12 198 0

S2 52.5 K+200 0

S1  10395  12K  2400 


 
 52.5 
S0 (K + 200)

 10395  12K  2400 


 For marginal stability (or) for poles lies on j axis  0
 52.5 
K = 666.25

ACE Engineering Academy Hyderabad|Delhi|Bhopal|Pune|Bhubaneswar| Lucknow|Patna|Bengaluru|Chennai|Vijayawada|Vizag |Tirupati | Kukatpally| Kolkata



: 28 : ECE

25. Two Ge diodes are connected in series opposition across a 5V battery as shown in figure.
Find the voltage across diode D1. Assuming breakdown voltage of diodes is greater than 5V.

– V1 + – V2 +

D1 D2

– +
5V

(A) 5V (B) 0V
(C) 0.018V (D) 4.982V
Ans: (D)
Sol: The current flowing through D1 and D2 is I0 as the diode D1 is in reverse biased

Consider D2:

I  I 0 e V / VT  1
I = I0 and V= V2

I 0  I 0 e V2 / VT  1 
V2 = VT ln(2) = 0.018V
V1 = 5 – 0.018 = 4.982V

Distractor Logic

DLOA: If we assume D1 is in complete reverse biased and open circuited then we will find this answer

DLOB: If we assume D1 is in complete forward biased and short circuited then we will find this answer

DLOC: If we assume D1 is in complete forward biased and I = I0 then we will find this answer

DLOD: True

ACE Engineering Academy Hyderabad|Delhi|Bhopal|Pune|Bhubaneswar| Lucknow|Patna|Bengaluru|Chennai|Vijayawada|Vizag |Tirupati | Kukatpally| Kolkata



: 29 : ECE

26. An ideal MOS capacitor maintained at T = 300k, has the following parameters:
 Gate material is P+ polycrystalline silicon.
 Substrate is n-type Si with doping concentration of 1018 cm–3. The flat band voltage of the
MOS capacitor is ____ volts.
Ans: 1.03 (Range: 0.9 to 1.1)
Sol: EVac EVac

qs q s

EC EC
EFS Eg/2
qBN
EI EI
|qVFB| Eg/2
EV, EFM EV,

qm = q  s + Eg

Eg
qs  q s   q BN
2
Eg
q m  qs   q BN
2
Eg
VFB   m  s    BN
2q
N   1018 
 BN  VT ln D   0.026 ln   0.47 V
10 
 ni   1.5  10 
E 1.12eV
VFB  g   BN   0.47  1.03V
2q 2q

27. Consider the following circuit. If the network is operating with 0.6 lagging p.f, determine RX
value and power dissipated by the circuit
Coil(1)

XL1  6 R1=2
+
V(t) =70.7sin377t RX
– XL2  2 R2=1

Coil(2)

ACE Engineering Academy Hyderabad|Delhi|Bhopal|Pune|Bhubaneswar| Lucknow|Patna|Bengaluru|Chennai|Vijayawada|Vizag |Tirupati | Kukatpally| Kolkata



: 30 : ECE

(A) 2, 150W (B) 2.5, 300W
(C) 3, 150W (D) None
Ans: (C)
Sol: V(t) = 70.7 sin(377t)
Coil (1) z1 = (2+j6)
Coil (2) z2 = (1+j2)
Total impedance z = z1 + z2 + Rx
z = (3+Rx+j8)
Pf = 0.6 = cos
10
8 8
tan   
6
6
8 8
tan     3+Rx = 6
3 Rx 6

Rx = 3

V 70.7 / 2 50
I    5A
Z 6  j8 10
P = I2 = (5)2 (2+1+3)
= 150W

Distractor Logic

DLOA: 2, 150 W


Output power correct P = 150 W
Rx = 2 is wrong

DLOB: 25, 300 W


Rx value 2.5 is wrong
Power = 2(I2R) = 300 W

DLOC: Correct option

DLOD: None

ACE Engineering Academy Hyderabad|Delhi|Bhopal|Pune|Bhubaneswar| Lucknow|Patna|Bengaluru|Chennai|Vijayawada|Vizag |Tirupati | Kukatpally| Kolkata



: 31 : ECE

28. The system of linear equations
x1 – 3x2 + 2x3 = 5
x1 – x3 = – 1
x1 – 2x2 + x3 = 5 has

(A) exactly two solutions (B) no solution


(C) a unique solution (D) many solutions
Ans: (B)
Sol: Consider the augmented matrix of the given system
1  3 2 5 
 
[A|B] = 1 0  1  1
1  2 1 5 
R2  R2 – R1, R3  R3 – R1
1  3 2 5 
 
~ 0 3  3  6 
0 1  1 0 
R3  3R3 – R2
1  3 2 5 
 
~ 0 3  3  6 
0 0 0 6 
Here, (A) = 2 and (A|B) = 3
 The given system is inconsistent and has no solution

29. Consider given op-amp is ideal and the transistors are matched.
The voltage Vx (in Volts) across resistor 1 k is _____ (given Vt = 25 mV and  is large)
+
5k Vx 1k
– +10V

+
V0
201.2
–10V
Q1
Q2

ACE Engineering Academy Hyderabad|Delhi|Bhopal|Pune|Bhubaneswar| Lucknow|Patna|Bengaluru|Chennai|Vijayawada|Vizag |Tirupati | Kukatpally| Kolkata



: 32 : ECE

Ans: 0.9998 (Range: 0.9 to 1.1)
V0  VBE1 VBE1  VBE 2
Sol: 
5k 201.2
V0
VBE
+
I C  ISe Vt
5k Vx 1k
– +10V
I  VBE1 –
VBE  Vt ln  C  V0
 IS  VBE1 +
201.2
–10V
VBE 2 Q1
I 
Similarly VBE1  Vt ln  C1  Q2
 IS 

I 
VBE 2  Vt ln  C 2 
 IS 

If transistors are matched


I 
VBE1  VBE 2  Vt ln  C1 
 IC2 
 Ic 
Vt ln  1 
V  I c2 
 x 
5k 201.2

V0  VBE 1
But I c1  ,
1k
V0  VBE 1
Ic 2 
5k
I c1 5k

Ic 2 1k

5k  5k 
 Vx  (25m) ln  
201.2  1k 
= 0.9998 V

ACE Engineering Academy Hyderabad|Delhi|Bhopal|Pune|Bhubaneswar| Lucknow|Patna|Bengaluru|Chennai|Vijayawada|Vizag |Tirupati | Kukatpally| Kolkata



: 33 : ECE

ACE Engineering Academy Hyderabad|Delhi|Bhopal|Pune|Bhubaneswar| Lucknow|Patna|Bengaluru|Chennai|Vijayawada|Vizag |Tirupati | Kukatpally| Kolkata



: 34 : ECE

30.
y0
y1
DEMUX y2 D1 Q1
1 y3 Clk FF
Enable y4
y5
y6 D0 Q0
y FF
s2 s1 s0 7

Input x

In the above state machine present state is Q1Q0 = 10


Find the next state if
(a) The input x = 1
(b) The input x = 0
(A) (a) 00 and (b) 11 (B) (a) 01 and (b) 11
(C) (a) 11 and (b) 01 (D) None of the above
Ans: (B)
Sol: (a) If x = 1  then s2s1s0 = 110 and Y6 = 1 and demux other outputs are 0’s
Then D1D0 = 01  Next state Q1Q0 = 01
(b) If x = 0  then s2s1s0 = 010 and Y2 = 1 and demux other outputs are 0’s
Then D1D0 = 11  Next state Q1Q0 = 11

Distractor Logic
DLOA: Next state are (a) 00 and (b) 11
If the input x is taken as LSB of demux then selection lines s2s1s0 values become 011 and 010
respectively

DLOB: Next state are 01 if x = 1, 11 if x = 0 → correct answer

DLOC: Next state (a) 11 and (b) 01


If s2 s1 are taken as next state values

DLOD: Not possible to determine as D1D0 Initial are not given

ACE Engineering Academy Hyderabad|Delhi|Bhopal|Pune|Bhubaneswar| Lucknow|Patna|Bengaluru|Chennai|Vijayawada|Vizag |Tirupati | Kukatpally| Kolkata



: 35 : ECE

2s  2
31. The transfer function of an LTI system is H(s)  . The steady state response of this
s  4s  4
2

system due to the input 4cos(2t) is observed to be kcos(2t+), then the value of ‘k’ is _______
Ans: 2.236 (Range: 2.0 to 2.5)
Sol: Given x(t) = 4cos(2t). So, 0 = 2 rad/sec

A cos(0t+) A|H(j0)|cos(0t+ +H(j0))


H(j)

2  2(2 j) 2  4j 1 2j 1 j
H(s) s  j0      = 0.559  26.6o
 j2 (2 j)  4( j2)  4
2
8j 4j 2 4

 Output y(t) = (4) (0.559) cos(2t – 26.6o)


y(t) = 2.236cos(2t–26.6o)
so, k = 2.236

32. A left circularly polarized wave travelling in the positive z-direction is received by an elliptically
polarized antenna whose reception characteristics near the main lobe is given by
EA = (2 â x  jâ y ) f(r,,), then the polarization loss factor is ______ dB.

Ans: –10
Sol: Wave is left circularly polarized
So EW = E0 â x  jâ y  e–jz

EA= 2â x  jâ y f(r,,).

â x  jâ y
Unit vector of the wave, ˆ W 
2
2â x  jâ y
Unit vector of the electric field of the receiving antenna, ˆ A 
5
2
2 â x  jâ y 2â x  jâ y 1
Polarization loss factor (PLF)  ˆ W .ˆ A  . 
2 5 10

1
PLF (dB) =10log    10dB
 10 

ACE Engineering Academy Hyderabad|Delhi|Bhopal|Pune|Bhubaneswar| Lucknow|Patna|Bengaluru|Chennai|Vijayawada|Vizag |Tirupati | Kukatpally| Kolkata



: 36 : ECE

33. Consider the following two-port network. The value of h12 parameter is ______
1 1:2

40

Ans: 0.5
Sol:
1:n
1 1:2

40

[T1] [T2] [T3]

A B   1 0

For transformer, ABCD Parameters matrix is    n
 C D  0 n 
By Using Cascade Connection

1 1  1  1 0
0  1
[T] = [T1] [T2] [T3]    2  
1
0 1  0  
2  40 

0.55 2
 
0.05 2
V1 = 0.55 V2 – 2I2
I1 = 0.05 V2 – 2I2
V1
h12 
V2 I1  0

2I2 = 0.05 V2
V1 = 0.55V2 – 0.05V2 = 0.5V2
V1
h12   0.5
V2

ACE Engineering Academy Hyderabad|Delhi|Bhopal|Pune|Bhubaneswar| Lucknow|Patna|Bengaluru|Chennai|Vijayawada|Vizag |Tirupati | Kukatpally| Kolkata



: 37 : ECE

34. The message signal m(t) has a bandwidth of 10(kHz), a power of 16(W) and maximum
amplitude of 6(Volts). It is desirable to transmit this message to a destination via a channel with
N0
80(dB) attenuation and additive white noise with two sided PSD  10 12 W / Hz  , and
2
achieve a SNR at the demodulator output of at least 50(dB). The required transmitter power (in
kilo Watts) if conventional AM with modulation index equal to 0.8 considered is _______

Ans: 909 (Range: 908 to 910)

Sol: For standard AM:


S S
    
 N 0  N b

 2 Pmn
Where,   (where Pmn = normalized message signal power)
1   2 Pmn

Pm 16 4
Pmn    (where Pm = message signal power)
max | m( t ) | 36 9
2

S PT  108
  
 N b WN 0

PT  10 8 P
 12
 T [Note ∵ PR = PT  attenuation = PT  10–8]
10  2 10
4
2

S
0.82  4 P
   9  T
N 2 4
  0 0.8   1 2
9
S P
   0.22  T  10
5

 N 0 2

 S  
   50(dB)
 N  0 

 PT ≃ 909 (kW)

ACE Engineering Academy Hyderabad|Delhi|Bhopal|Pune|Bhubaneswar| Lucknow|Patna|Bengaluru|Chennai|Vijayawada|Vizag |Tirupati | Kukatpally| Kolkata



: 38 : ECE

35. For the circuit shown, VBE = 0.7V for Q2 at 1mA, Vt = 25mV (thermal voltage). The value of R
(in k) that will result in I0 = 10A is ____ (Assume all transistors are identical with large )

10A I0

Q1 Q3

Q2 R

Ans: 58.48 (Range: 57 to 60)


Sol: Let VBEX be the voltage when current IX = 10A is passing through Q2

Let VBEY be the voltage when current IY = 1mA is passing through Q2

I
VBE = Vt ln  
 Is 

I 
VBEX  VBEY  Vt ln  1  [in general]
 I2 
10 
VBEX – 0.7 = 25(mV) ln 
 1m 
VBEX = 0.5848V = VBE 2

From the problem


KVL:
– VBE 2 – VBE1 + VBE 3 + I0R = 0

VBE1 = VBE 2 = VBE 3

VBE
R 
I0
0.5848
  58.48k
10

ACE Engineering Academy Hyderabad|Delhi|Bhopal|Pune|Bhubaneswar| Lucknow|Patna|Bengaluru|Chennai|Vijayawada|Vizag |Tirupati | Kukatpally| Kolkata



: 39 : ECE

36. In a Si sample, the acceptor energy level is 0.16eV above the valance band offset (Ev). The Fermi
energy level is 0.26eV above Ev at 3000K. If the acceptor doping concentration is NA = 1016/cm3.
Find the concentration of un-ionized acceptor atoms.

(A) 1016 /cm3 (B) 0.9791016 /cm3


(C) 0.0211016 /cm3 (D) 1.9791016 /cm3
Ans: (C)
Sol: Given EF – EV = 0.26eV, Ea – EV = 0.16eV
T = 3000K and NA = 1016/cm3

According to Fermi-Dirac distribution


1
f (E)  (E  E F )/KT
1 e

At acceptor energy level Ea


1 1 1
f (E a )  (Ea  E F )/ KT
 (E a  E V )  (E F  E V )/KT
 0.160.26  / KT
1 e 1 e 1 e
= 0.979 (Assuming KT = 0.026 eV)

Concentration of un-ionized atoms


= (1 – f(Ea)). NA
= 0.0211016 /cm3

Distractor Logic

DLOA: If we had considered only NA as it is for the whole calculation

DLOB: If we had taken the formula for concentration of un-ionized atoms as f(Ea)NA, then we might
have got this answer

DLOC: True

DLOD: If we had taken the formula for concentration of un-ionized atoms as (1+f(Ea)NA), then we
might have got this answer

ACE Engineering Academy Hyderabad|Delhi|Bhopal|Pune|Bhubaneswar| Lucknow|Patna|Bengaluru|Chennai|Vijayawada|Vizag |Tirupati | Kukatpally| Kolkata



: 40 : ECE

37. The directional derivative of (x, y, z) = xy2z + 4yz2 at the point P(1, 2, –1) along i  2 j  k is
1
(A) –16 (B)
6
 16 16
(C) (D)
6 7
Ans: (C)
Sol: Given that (x, y, z) = xy2z + 4yz2
Let a  i  2 j  k & P = (1, 2, –1)
  
Now  = grad  = i  j k
x y x

  
  = i y 2 z  j 2xyz  4z 2  k xy 2  8 yz  
 ()P = (–4) i + (0) j + (–12) k

Now the directional derivative of the function (x, y, z) in the direction of a vector a at the point

P = ()P .
a
=  4  i  0  j   12  k .
i  2 j  k  =
 41  0 2   121
|a| 1   2  1
2 2 2
1 4 1

 16
=
6

2s  1
38. An analog filter transfer function H(s)  is to be converted to digital filter sampled at
s  5s  4
2

2Hz using impulse-invariant method. The digital filter transfer function observed to be
2z 2  1.6843z
H(z)  . Then ‘’ is _________
z 2  z  0.0831

Ans: 0.7415 (Range: 0.7 to 0.8)

2s  1 3 1
Sol: H(s)   
(s  1)(s  4) s  4 s  1

1 1
Using impulse-invariant method  Ts 1
s 1 e z

ACE Engineering Academy Hyderabad|Delhi|Bhopal|Pune|Bhubaneswar| Lucknow|Patna|Bengaluru|Chennai|Vijayawada|Vizag |Tirupati | Kukatpally| Kolkata



: 41 : ECE

1
Given Ts = = 0.5 sec
2
3 1
H (z)  ( 4 )( 0.5 ) 1
 (1)( 0.5 ) 1
1 e z 1 e z
3 1 3  1.819z 1  1  0.1357z 1
H(z)   
1  0.135z 1 1  0.6065z 1 1  0.7415z 1  0.0818z  2
2  1.684z 1

1  0.7415z 1  0.0818z  2
2z 2  1.6843z
 so,  = 0.7415
z 2  0.7415z  0.0818

K S  5
39. A unity feedback control system has an OLTF G S  . The root locus of complex roots
S  12
are part of a circle. The radius of the circle is _______.
Ans: 4
Sol: To get radius of circle, we required break points.
dK
Break point are obtained by 0
dS
CE  1  G S  0
K S  5
1 0
S  12
 S  1  K S  5  0
2

K

 S2  2S  1 
S  5



dK  2S  2S  5  S2  2S  1
 0
 
dS S  52
 –[2S2 + 2S + 10S + 10 – S2 – 2S – 1] = 0
 –[S2 + 10S + 9] = 0
(S+1)(S+9) = 0  Break points are = –1, –9
1 9
Centre   5, (mid point of Break points)
2
ACE Engineering Academy Hyderabad|Delhi|Bhopal|Pune|Bhubaneswar| Lucknow|Patna|Bengaluru|Chennai|Vijayawada|Vizag |Tirupati | Kukatpally| Kolkata

: 42 : ECE

Centre (–5, 0)
Radius is distance between centre to breaks points.

BP –5 BP
–9 –1
r =4 r =4
Radius = 4

40. Three different dielectrics of permittivities 40, 90, and 30 are defined in the space as shown
in figure. If the uniform plane wave is incident on the plane x = –6m, then the time it will take to
strike the interface between dielectric (2) and dielectric (3) is ____ nano second.

Dielectric Dielectric Dielectric


UPW k̂  â free (1) (2) (3)
x
space 0,40 0,90 0,30

x= – 6m x=0 x=3m x=5m


Ans: 60
Sol: vp1 = Phase velocity of the uniform plane wave in free space
vp2 = Phase velocity of the uniform plane wave in dielectric (1)
vp3 = Phase velocity of the uniform plane wave in dielectric (2)
vp1 = 3108 m/s
3  108 3
vP 2    108 m / s
r 2

3  108
v P3   1  108 m / s
r

t1 = time taken by the uniform plane wave to travel in free space


t2 = time taken by the uniform plane wave to travel in dielectric (1)
t3 = time taken by the uniform plane wave to travel in dielectric (2)
ACE Engineering Academy Hyderabad|Delhi|Bhopal|Pune|Bhubaneswar| Lucknow|Patna|Bengaluru|Chennai|Vijayawada|Vizag |Tirupati | Kukatpally| Kolkata

: 43 : ECE

6
t1  =210–8 sec =20ns
3  108

3
t2  =210–8 sec =20ns
3
 108
2
2
t3  =210–8 sec =20ns
1  108

Total time taken to strike the interface between 2& 3 is


T = t1 + t2 + t3 = 20 + 20 + 20 = 60ns

41. An abrupt Si pn-junction with NA = 1018cm–3 and ND = 1017 cm–3 is maintained at T = 300K.
The minority carrier lifetimes in the p-side and n-side are n = 10–8s and p =10–17 s, respectively.
The concentration of holes injected at the edge of depletion layer on the n-side when a forward
bias of 0.6V is applied will be _____ 1013 cm–3 [Assume VT = 26mV at T = 300 K]
Ans: 2.37 (Range: 2.20 to 2.50)
Sol: ND = 1017 cm–3  n n 0

n i2 2.25  10 20
Pn 0   17
= 2.25  103 cm–3
n n0 10

Pne  Pn 0 e VF VT

Concentration of injected carriers at X = Xn = Pne – Pn0



 Pn 0 e VF VT
1 
 2.37  1013 cm 3

42. Identify the voltage transfer characteristics (VTC) of the given inverting Schmitt trigger
circuit, assuming VD(ON) = 0.7V and  Vsat =  6.7V. Assume Op-Amp is ideal
6.7V
– V0 D1
+ +
Vin – 1k
–6.7V D2 1k
1k
1k

ACE Engineering Academy Hyderabad|Delhi|Bhopal|Pune|Bhubaneswar| Lucknow|Patna|Bengaluru|Chennai|Vijayawada|Vizag |Tirupati | Kukatpally| Kolkata



: 44 : ECE

(A) V0 (B) V0
+6.7V 6.7V

–3V +3V Vi Vi
–3.35V 3.35V

–6.7V –6.7V

(C) (D) V0
V0
6.7V
+6V
Vi
–2V +2V
–2V +2V Vi
–6.7V
–6V

Ans: (D)
Sol: Assume V0 = +6.7V
D1 (6V)
D1 FB, D2 RB 6.7V
1k
6(0.5k ) +
VP   2V
1k  0.5k
1k 1k VP

If Vin > 2V then V0 switches to –6.7V
D2 (–6V)
–6.7V
1k
+
1k 1k VP

Assume V0 = –6.7V D1 RB, D2 FB


 6(0.5k )
VP   2V
1k  0.5k

ACE Engineering Academy Hyderabad|Delhi|Bhopal|Pune|Bhubaneswar| Lucknow|Patna|Bengaluru|Chennai|Vijayawada|Vizag |Tirupati | Kukatpally| Kolkata



: 45 : ECE

Distractor Logic

DLOA: If student neglects 1k resistor,


D1
6V
1k
+
1k 3V
VUTP = +3V,

Similarly
VLTP = –3V.

DLOB: Students who assume diode as ideal


VUTP = 3.35V
D1
6.7 V
1k
+
1k VUTP = 3.35V
Similarly

VLTP = –3.35V.

DLOC: Students who calculate correctly, but mismarking the saturation limit as 6V instead of 6.7V

DLOD: Correct answer

43. The given ideal ammeter reading magnitude, if it reads RMS value in Amperes, is ______
(Given voltage source values are RMS values)
12

– A
–j2 100900
+
I1 I2 4

+ 0
– 100–90 –j4
I3

j3
ACE Engineering Academy Hyderabad|Delhi|Bhopal|Pune|Bhubaneswar| Lucknow|Patna|Bengaluru|Chennai|Vijayawada|Vizag |Tirupati | Kukatpally| Kolkata

: 46 : ECE

Ans: 25
Sol: Ammeter RMS value is I2 12

– A
–j2 100900
+
I1 I2 4
By KVL for I2 loop
+ 0
(12+4)I2 – j4(I2–I3) + 10090 = 0 – 100–90 –j4
(16–j4)I2 + j4I3 + j100 = 0……… (1) I3

By KVL for I3 loop j3


100–90 = –j4(I3–I2) +j3I3
–j100 = j4I2 – jI3
I3 = (100+4I2)
From (1)
(16–j4)I2 + j4(100+4I2) + j100 = 0
I2 (16+12j) = –j500
 j500
I2 
16  12 j
500
I2   25Amps
20

44. For the system shown in figure, the peak overshoot and percentage of peak overshoot to unit step
input are

R(S) + 20 C(S)
– S  4S  5
2

(A) 0.00756, 0.189%


(B) 0.00189, 0.189%
(C) 0.203, 25.38%
(D) 0.253, 25.38%

ACE Engineering Academy Hyderabad|Delhi|Bhopal|Pune|Bhubaneswar| Lucknow|Patna|Bengaluru|Chennai|Vijayawada|Vizag |Tirupati | Kukatpally| Kolkata



: 47 : ECE

Ans: (C)
CS  20 
Sol: CLTF   2 
R S  S  4S  25 
20  25 
  2 
25  S  4S  25 
20
n = 5 rad/sec,  = 0.4, k 
25
  / 1  2  20  0.4 / 1 0.4 2 
M p  Ak e  
 1 e 
 0.203
25
  / 1  2    0.4 / 1 0.4 2 
Mp  e  
 100%  e  
 100%  25.38%

Distractor Logic

CS  20 
DLOA: If TF,  2  , then
R S  S  4S  5 
Mp = 0.00756
% Mp = 0.189%

DLOB: If closed loop system gain is 1 and CE is S2 + 4S + 5 = 0


then Mp = 0.00189
% Mp = 0.189%

 20 
DLOC: Correct option CLTF   2 
 S  4S  25 
Mp = 0.203
% Mp = 25.38%

DLOD: If closed loop system gain is 1 and CE: S2 + 4S + 25 = 0


then Mp = 0.253
% Mp = 25.38%

ACE Engineering Academy Hyderabad|Delhi|Bhopal|Pune|Bhubaneswar| Lucknow|Patna|Bengaluru|Chennai|Vijayawada|Vizag |Tirupati | Kukatpally| Kolkata



: 48 : ECE

45. In a 4-bit ripple carry adder, the full adder takes 3ns to produce sum, 2ns to produce carry.
Initially all the values are cleared. When inputs are hexadecimal numbers A = F and B = 1, what
is the value of adder output in Hexadecimal at 7ns.
(A) 0 (B) 8
(C) E (D) C
Ans: (B)
Sol: To generate final output it takes 9ns. So at 7ns FA4 will not stabilize.
At 7ns

0 1 0 1 0 1 1 1

FA4 0 FA3 1 FA2 1 FA1

0 1 0 0 0

So FA4 will add only B3 and A3. So the output is 1000 = 8H

Distractor Logic

DLOA: 0H = 00002
If the sum is computed then FH + 1H = 0H with cy = 1.

DLOB: 8H = 10002 correct

DLOC: EH = 11102
And consider that only 1 output bit is changed then
If they perform addition as  1 1 1 1
+1
1 1 1 0
DLOD: CH = 11002
After addition if they consider that first two output bits are changed

ACE Engineering Academy Hyderabad|Delhi|Bhopal|Pune|Bhubaneswar| Lucknow|Patna|Bengaluru|Chennai|Vijayawada|Vizag |Tirupati | Kukatpally| Kolkata



: 49 : ECE

ACE Engineering Academy Hyderabad|Delhi|Bhopal|Pune|Bhubaneswar| Lucknow|Patna|Bengaluru|Chennai|Vijayawada|Vizag |Tirupati | Kukatpally| Kolkata



: 50 : ECE

46. A communication channel transmits, two messages m1 and m2 in random order. The message m1
occurs three times more frequently than m2. Message m1 generates receiver response r1 = –1(V)
while m2 generates r2 = +1(V). The channel is corrupted by noise ‘n’ with uniform pdf which
extends from n = –1.5V to n = 1.5V. If the threshold voltage at the receiver is taken as zero volts
then the probability of error Pe will be

1 1
(A) (B)
2 6
1 1
(C) (D)
4 3
Ans: (B)
3 1
Sol: P(m1 )  , P(m 2 ) 
4 4
Given noise is uniformly distributed between [–1.5(V) to 1.5(V)]
f(n) =1/3

–1.5 0 1.5
n in V

noise effects signal which results in Error

Noise algorithm:
At comparator input if m 1  n v  0  [m 1 is decoded as m 2 ]
–1(V) + nv > 0
1/3 Pe (m 2 /m1 )
n v  1(V)

n(V)
0.5 –1.5 0 1V 1.5
Pe (m 2 /m1 ) 
3

At comparator input m 2  n v  0  [m 2 is decoded as m 1 ]


1(V) + nv < 0
Pe (m1/m 2 ) 1/3
n v  1(V)

n(V)
–1.5 –1V 0 1.5

ACE Engineering Academy Hyderabad|Delhi|Bhopal|Pune|Bhubaneswar| Lucknow|Patna|Bengaluru|Chennai|Vijayawada|Vizag |Tirupati | Kukatpally| Kolkata



: 51 : ECE

0.5
Pe (m1/m 2 ) 
3

 Pe  P m1  Pm1 decoded as m 2   P m 2  Pm 2 decoded as m1 

3  0.5  1  0.5  1
Pe       
4 3  4 3  6

Distractor Logic

DLOA: Is wrong answer


Considering the threshold voltage as zero, if student simply takes the area under the noise pdf
curve then without using noise algorithm
Vth

1/3

3 1 1  1
Pe  1.5    1.5    n(V)
4 3 4  3 –1.5 0 1.5

3 1 1 1
Pe    
4 2 4 2
1
Pe 
2

DLOB: Is correct answer


DLOC: Wrong answer
Students makes a mistake by considering noise pdf in correctly as then even if he does the noise
algorithm correctly then

1/2

–1.5 0 1.5

3  0.5  1  0.5 
Pe  
4  2  4  2 
1
Pe 
4
ACE Engineering Academy Hyderabad|Delhi|Bhopal|Pune|Bhubaneswar| Lucknow|Patna|Bengaluru|Chennai|Vijayawada|Vizag |Tirupati | Kukatpally| Kolkata

: 52 : ECE

DLOD: Wrong answer
If student does the noise algorithm correctly but forgets to take the P(m1) and P(m2) then
0.5 0.5
Pe  
3 3
1 1 1
Pe   
6 6 3

47. The CV curve of a MOS capacitor of area 100m100m is shown below:

C(pF)

70

VGB(V)
–0.25 V 0.7 V

What is the type of substrate and the thickness of the oxide in nm ?


(Where  ox  3.5  10 13 F / cm )
(A) P-type and tox = 5 nm (B) N-type and tox = 500 nm
(C) P-type and tox = 500 nm (D) N-type and tox = 5 nm
Ans: (D)
Sol: From the CV plot
Cmax = A.Cox = 70pF
A.Cox = 70pF
 ox
A.  70pF
t ox
A. ox
t ox 
70pF


100 10   3.5 10
4 2 13

70  10 12
= 5  10–7 cm
= 5nm
ACE Engineering Academy Hyderabad|Delhi|Bhopal|Pune|Bhubaneswar| Lucknow|Patna|Bengaluru|Chennai|Vijayawada|Vizag |Tirupati | Kukatpally| Kolkata

: 53 : ECE

Distractor Logic

DLOA: If we think it is P-type

DLOB: If we would have taken the final answer to be in SI units instead of CGS

DLOC: If we would have taken the final answer to be in SI units instead of CGS

DLOD: Correct option

W
48. Given transconductance parameter k =  n C ox  1 mA / V 2 . If the transistors are matched, then
L
the value of V0 is (in Volts) ______
+10V

Q4 Q3

V0
Q1 Q2

Ans: 5
Sol: VG 2  VD 2

(VGS2  Vt ) 2
 Q2 is in saturation I D 2  k
2

VGS2  VGS3 (∵ I D 2  I D3 )

From the figure given in the question VGS2  VGS1

VGS1  VGS4 ( ∵ I D1  I D 4 )

 VGS1  VGS2  VGS3  VGS4

10 = VGS3  VGS2

10 = 2 VGS2 ( ∵ VGS2  VGS3 )

ACE Engineering Academy Hyderabad|Delhi|Bhopal|Pune|Bhubaneswar| Lucknow|Patna|Bengaluru|Chennai|Vijayawada|Vizag |Tirupati | Kukatpally| Kolkata



: 54 : ECE

VGS2  5V
VGS4  5V
V0  10  VGS 4
V0  5V

49. A linear time invariant system is characterized by the homogeneous state equations
  0 1   x1 
 x 1     
 x 2   1  2  x 2 

The state transition matrix is


1  t e  t  te  t  1  t e  t te  t 
(A)  (B) 
 te
t
1  t e t    te
t
1  t e  t

 1  t e  t te  t  1  t e  t te  t 
(C)  (D) 
  te
t
 1  t e t    te
t 
1  te t 

Ans: (B)
 Adj.SI  A 
Sol: State transition matrix = eAt  L1 SI  A   L1 
1
 
 SI  A 
0 1 S  1 
A   SI  A    
  1  2 1 S  2
 S 2 1   S 2 1 
   S  12
e At  L1  S  2S  1 S  2S  1  L1 
2 2 S  12   1  t e t 
te t
1 S  1 S    te t 1  t e  t
 2 
 S  2S  1 S  2S  1
2
 S  12 S  12 
Distractor Logic
 S 1 
 S  12 S  12  , then eAt become option A
DLOA: If AdjSI  A   
 1 S 2 
 S  12 S  12 
DLOB: Correct Option

DLOC: If partial fraction calculations are wrong, then option C is possible.

DLOD: If Partial fraction calculations are wrong, then option D is possible.


ACE Engineering Academy Hyderabad|Delhi|Bhopal|Pune|Bhubaneswar| Lucknow|Patna|Bengaluru|Chennai|Vijayawada|Vizag |Tirupati | Kukatpally| Kolkata

: 55 : ECE

50. A signal x(t) (band limited to a frequency of fm(Hz)) is multiplied by a rectangular pulse c(t) as
shown in the figure below.

c(t)

0.2510–3
1

 t(s)
10–3 0
What must be the value of ‘fm’ (in Hz) that has to be ensured so that x(t) can be recovered from
the product x(t) c(t) obtained using an ILPF (Ideal Low pass filter)?

(A) fm 1000(Hz) (B) fm 500(Hz)


(C) fm  500(Hz) (D) fm  1000(Hz)
Ans: (B)
Sol: Consider s(t) = x(t) c(t)
x(t) is a band limited signal whose maximum frequency is ‘fm’ (Hz)
x ( t ) CTFT
 X(f )
X(f)
1


–fm 0 fm f (Hz)

c(t) is a periodic train of square pulses whose period is 10–3 (Sec)



1
ct  CTFT
 Cf    Cn / T f  n / T 
0 0
T0 n  


1  n   n 
 Cf    0.25 10 3
sin c  0.25  10 3 3  f  3 
10 3 n    10   10 

 

Cf   0.25  sin c0.25n  f  n  10 3
n  

Since s(t) = x(t) c(t)


S(f) = X(f) C(f)
ACE Engineering Academy Hyderabad|Delhi|Bhopal|Pune|Bhubaneswar| Lucknow|Patna|Bengaluru|Chennai|Vijayawada|Vizag |Tirupati | Kukatpally| Kolkata

: 56 : ECE

 
 

S(f) =X(f) 0.25  sin cn / 4 f  n  10 3 
 n   

 

Sf   0.25  sin c n / 4X f  n 103
n  

 X f  103   .........
2 2
Sf   0.25  1  Xf   0.25

 

at n  0
 
at n 1

NOTE: S(f) = 0 at Integer multiples of 4

S(f)
0.25 2 2
0.25 

Difference>0
–fm fm 103–fm 103 103+fm f (Hz)
0

For reconstruction of x(t) from the product x(t) c(t) we require


103 – fm – fm  0
2fm  103
fm  500 (Hz)
 Option (B) is correct

Distractor Logic

DLOA: Is wrong answer


If student does the problem correctly but at the last step if he considers
103 – fm  0
fm  1000 (Hz)

DLOB: Is correct answer

ACE Engineering Academy Hyderabad|Delhi|Bhopal|Pune|Bhubaneswar| Lucknow|Patna|Bengaluru|Chennai|Vijayawada|Vizag |Tirupati | Kukatpally| Kolkata



: 57 : ECE

DLOC: Is wrong answer
At the last step if student considers
103 – fm – fm  0
2fm  1000
fm  500 (Hz)

DLOD: Is wrong answer


At the last step if student considers
103 – fm  0
103  fm
fm  1000 (Hz)

51. If the Nyquist rate for x(t) is s, then find the Nyquist rate of x2(2t).

(A) 4s (B) 2s


(C) 8s (D) s

Ans: (A)
s
Sol: From the given data the maximum frequency of x(t) is
2
X()


 s
 s
2 2

1  
x (2t )  X 
2 2
 
X 
2


–s s

ACE Engineering Academy Hyderabad|Delhi|Bhopal|Pune|Bhubaneswar| Lucknow|Patna|Bengaluru|Chennai|Vijayawada|Vizag |Tirupati | Kukatpally| Kolkata



: 58 : ECE

Nyquist rate of x(2t) is 2s
y( t )  x 2 ( 2 t )  x ( 2 t ) x ( 2 t )

1  1    1   
Y()  X    X 
2  2  2  2  2 

1      
Y()   X   X  
8   2   2 

Y()


–2s 2s

Nyquist rate of x2(2t) is 22s = 4s

Distractor Logic

DLOA: Correct option

s
DLOB: While drawing the spectrum of X() if we consider maximum frequency as , then
4
option (B) is correct.

DLOC: While drawing the spectrum of X() if we consider maximum frequency as is s , then

option (C) is correct.

DLOD: If we think that the Nyquist Rate of x(t) and x2(2t) is same then option (D) is correct.

ACE Engineering Academy Hyderabad|Delhi|Bhopal|Pune|Bhubaneswar| Lucknow|Patna|Bengaluru|Chennai|Vijayawada|Vizag |Tirupati | Kukatpally| Kolkata



: 59 : ECE

52. The result of a measurement of the voltage standing wave pattern on a transmission line with
characteristic impedance Z0 = 100 is shown in figure.
V ( )

Vmax=4

Vmin=1

 0
8cm 1
cm

The value of load impedance, ZL is ________.

(A) 46.94 + j88.29 (B) 400


(C) 125.2 + j166 (D) 28.98 – j38.43
Ans: (C)
Sol: Correct option:
From the given figure

 8cm   = 16cm
2
L
 Vmax1 
4
L 
1cm 
4
(1cm) 4 180
L 
16cm
 L = 45o …………(1)
Vmax
 VSWR  4
Vmin

1  L
 VSWR  4 
1  L
ACE Engineering Academy Hyderabad|Delhi|Bhopal|Pune|Bhubaneswar| Lucknow|Patna|Bengaluru|Chennai|Vijayawada|Vizag |Tirupati | Kukatpally| Kolkata

: 60 : ECE

4  4 L  1  L

3  5 L

3
 L   0.6 ………(2)
5

From (1) and (2)


0.6
1  j = 0.424 (1 + j)
o
L  L e j L  0.6 e j45 
2

1  L  1  0.424  j 0.424   1.424  j 0.424   0.576  j 0.424 


Z L  Z0    100    100    
1  L  1  0.424  j 0.424   0.576  j 0.424   0.576  j 0.424 
 0.64 0.848 
 100 j  1001.252  j1.66
 0.511 0.511 
Z L  125.2  j166 

Distractor Logic
DLOA: If they take the distance from Vmax to Vmax as ‘’ then  = 8cm,
 L  1cm  4  180 o
 Vmax1  =  L
4 8cm 
 L = 90o
VSWR = 4,  L  0.6
o
 L  0.6 e j90  j 0.6
L  j 0.6

 1  j 0 .6   130.96 
 Z L  100    100  

 1  j 0 .6   329 
 100  298o  = 46.94 + j88.29

DLOB: If they use VSWR to calculate L then L = 0.6


1  0.6 
and Z L  100   400
1  0.6 
ZL = 400
ACE Engineering Academy Hyderabad|Delhi|Bhopal|Pune|Bhubaneswar| Lucknow|Patna|Bengaluru|Chennai|Vijayawada|Vizag |Tirupati | Kukatpally| Kolkata

: 61 : ECE

DLOC: Correct Option
DLOD: If they consider
 L  2m  1  
 Vmax1   , instead of  Vmax1  L
4 4 4
L 
 Vmax1   (form = 0)
4 4

 L (16cm)   
 8cm 
1cm   4cm  2 
4  180 o   16cm


 3cm   4  180 o
 L
16cm 
  L  135o

From VSWR = 4, L  0.6

 L  0.6 e j135  = 0.6 [–0.707 – j0.707]


o

L  0.424 1  j

 0.576  j 0.424 
Z L  100    1000.4814 307.02
 1.424  j 0.424 
ZL = 28.98 – j38.43

d2y dy
53. The particular integral of 2x 2 2
 3x  3y  x 3 is ________.
dx dx
x3 x3
(A) (B)
6 12
x3 x3
(C) (D)
16 18

Ans: (D)
Sol: The given Euler-Cauchy's form can be converted to
d
2D(D – 1)y + 3Dy – 3y = e3z (where D = , x = ez & z = log x)
dz
ACE Engineering Academy Hyderabad|Delhi|Bhopal|Pune|Bhubaneswar| Lucknow|Patna|Bengaluru|Chennai|Vijayawada|Vizag |Tirupati | Kukatpally| Kolkata

: 62 : ECE

(2D2 + D – 3)y = e3z
e 3z
 yp =

2D 2  D  3 
e3z
=
2D  3D  1
e3z e3z x3
= = =
6  33  1 18 18
 Correct answer is option (D)

54. For the following circuit if X, Y are initially at 0 and 1 respectively. A square pulse is given at
input ‘X’ which has a transition of ‘0’ to ‘1’ at t = 10 ns having a pulse width of 80ns. The
propagation delays of NOT, AND gates are 5ns, 10ns respectively. Then the number of
transitions at the output ‘Z’ are ________ .

Y Z

Ans: 6
Sol:

X
10 90
Y

25 40 55 70 85 100

Z
20 35 50 65 80 95

The number of transitions at output ‘Z’ = 6

ACE Engineering Academy Hyderabad|Delhi|Bhopal|Pune|Bhubaneswar| Lucknow|Patna|Bengaluru|Chennai|Vijayawada|Vizag |Tirupati | Kukatpally| Kolkata



: 63 : ECE

55. By applying Newton-Raphson method, the second approximation of the required root of the
equation (x3 – 5x + 3) = 0 with initial guess x0 = 1 is _______.
Ans: 0.6471 (Range: 0.64 to 0.65)
Sol: Let f(x) = (x3 – 5x + 3)
f (x) = (3x2 – 5)

From N-R method,


f x 0 
x1 = x0 –
f x 0 

=1–
 1 = 1 – 0.5 = 0.5
2

f x 1 
 x2 = x1 –
f x1 

0.625
= 0.5 – = 0.6471
 4.25

ACE Engineering Academy Hyderabad|Delhi|Bhopal|Pune|Bhubaneswar| Lucknow|Patna|Bengaluru|Chennai|Vijayawada|Vizag |Tirupati | Kukatpally| Kolkata



: 64 : ECE

ACE Engineering Academy Hyderabad|Delhi|Bhopal|Pune|Bhubaneswar| Lucknow|Patna|Bengaluru|Chennai|Vijayawada|Vizag |Tirupati | Kukatpally| Kolkata

Das könnte Ihnen auch gefallen